Retirement Planning Flashcards

1
Q

What are the Qualified Retirement Plan types?

A
How well did you know this?
1
Not at all
2
3
4
5
Perfectly
2
Q

Which of the following is not a qualified retirement plan?

a) ESOP.
b) 401(k) plan.
c) 403(b) plan.
d) Target benefit plan.

A

Answer: C

A 403(b) plan is a tax-advantaged plan, not a qualified plan. All of the others are qualified plans.

How well did you know this?
1
Not at all
2
3
4
5
Perfectly
3
Q

Which of the following is an example of a qualified retirement plan?

a) Rabbi trust.
b) 401(k) plan.
c) Nonqualified stock option plan.
d) ESPP.

A

Answer: B

A 401(k) plan is a qualified plan. All of the others are not qualified retirement plans.

How well did you know this?
1
Not at all
2
3
4
5
Perfectly
4
Q

What are the differences between Pension Plans and Profit-Sharing Plans?

A
How well did you know this?
1
Not at all
2
3
4
5
Perfectly
5
Q

What are the differences between Defined Benefit Plans and Defined Contribution Plans ?

A
How well did you know this?
1
Not at all
2
3
4
5
Perfectly
6
Q

Each of the following is a characteristic of a defined benefit retirement plan EXCEPT:

a) The plan specifies the benefit an employee receives at retirement.
b) The law specifies the maximum allowable benefit payable from the plan is equal to the lesser of 100% of salary or $230,000 (2021) per year currently.
c) The plan has less predictable costs as compared to defined contribution plans.
d) The plan assigns the risk of pre-retirement inflation, investment performance, and adequacy of retirement income to the employee.

A

Answer: D

Option D describes characteristics of a defined contribution plan. Defined benefit plans assign the risk of pre-retirement inflation, investment performance, and adequacy of retirement income to the employer, not the employee.

How well did you know this?
1
Not at all
2
3
4
5
Perfectly
7
Q

What are the advantages of Qualified Retirement Plans?

A
How well did you know this?
1
Not at all
2
3
4
5
Perfectly
8
Q

The following statements concerning retirement plan service requirements for qualified plans are correct EXCEPT:

a) The term “year of service” refers to an employee who has worked at least 1,000 hours during the initial 12-month period after being employed.
b) If an employee hired on October 5, 20x1 has worked at least 1,000 hours or more by October 4, 20x2, he has acquired a year of service the day after he worked his 1,000th hour.
c) An employer has the option of increasing the one-year of service requirement to 2 years of service.
d) Once an employee attains the service requirement of the plan, the employer cannot make the employee wait more than an additional six months to be considered eligible to participate in the plan.

A

Answer: B

Option B is false because the employee would NOT acquire a year of service the day after he worked his 1,000th hour, but after twelve months AND 1,000 hours. While option C is correct, the exception does not apply to 401(k) plans.

How well did you know this?
1
Not at all
2
3
4
5
Perfectly
9
Q

Which of the following statements are reasons to delay eligibility of employees to participate in a retirement plan?

  1. Employees don’t start earning benefits until they become plan participants (except in defined benefit plans, which may count prior service).
  2. Since turnover is generally highest for employees in their first few years of employment and for younger employees, it makes sense from an administrative standpoint to delay their eligibility.

a) 1 only.
b) 2 only.
c) Both 1 and 2.
d) Neither 1 nor 2.

A

Answer: C

Both 1 and 2 are correct.

How well did you know this?
1
Not at all
2
3
4
5
Perfectly
10
Q

MAD Incorporated sponsors a qualified plan that requires employees to meet one year of service and to be 21 years old before being considered eligible to enter the plan. Which of the following employees are not eligible?

  1. Jim, age 18, who has worked full-time with the company for 3 years.
  2. Rachel, age 22, who has worked full-time with the company for 6 months.
  3. Brian, age 62, who has worked 500 hours per year for the past 6 years.
  4. Patrick, age 35, who has worked full-time with the company for 10 years.

a) 4 only.
b) 1 and 2.
c) 3 and
4. d) 1, 2, and 3

A

Answer: D

MAD can exclude anyone who has not attained age 21 and has not completed one year of service with the company with 1,000 hours during that year. Jim is not yet 21. Rachel has not completed a full 12 months of service. Brian does not work at least 1,000 hours each year.

How well did you know this?
1
Not at all
2
3
4
5
Perfectly
11
Q

Which of the three employees are considered highly compensated based on the qualified plan rules?

a) Elizabeth
b) Carol and David
c) Elizabeth, Carol and David
d) David

A

Answer: C

All three individuals are considered highly compensated for the current year. Even though Elizabeth does not own more than 5% in the current year, she is considered highly compensated because she owned greater than 5% in the proceeding plan year. Carol is highly compensated because she is a greater than 5% owner in the current year. David is highly compensated because his income exceeds the compensation limit ($130,000)for the current year.

How well did you know this?
1
Not at all
2
3
4
5
Perfectly
12
Q

What are the characteristics of highly compensated employees?

A
How well did you know this?
1
Not at all
2
3
4
5
Perfectly
13
Q

Which of the following people would be considered highly compensated for 2021 (assume the company made the election to reduce)?

a) Renee, a one percent owner who earns $80,000 per year.
b) Hannah, who earned $125,000 last year and is the 40th highest-paid employee of 100 employees.
c) Conrad, a 20 percent owner who earns $40,000 per year.
d) Daniel, a 5 percent owner who earns $45,000 per year.

A

Answer: C

Only Conrad would be considered highly compensated because he is greater than a 5 percent owner. An individual is deemed highly compensated if he is either a greater than 5 percent owner, or has earnings in excess of $130,000 (2021) and is in the top 20 percent, as ranked by salary of all employees.

How well did you know this?
1
Not at all
2
3
4
5
Perfectly
14
Q

ABC Co. has 125 employees. One hundred of these employees are nonexcludable and 25 of those are highly compensated (75 are nonhighly compensated). The company’s qualified profit sharing plan benefits 21 of the highly compensated employees and 55 of the nonhighly compensated employees. Does the plan meet the safe harbor coverage test?

a) No, percent covered is 50%.
b) Yes, percent covered is 73.3%.
c) No, percent covered is 73.3%.
d) Yes, percent covered is 50%.

A

Answer: B

The profit-sharing plan meets the general safe harbor coverage test because it benefits 73.33 percent (55 / 75) of the nonhighly compensated eligible employees.

Twenty (75-55) of the nonexcludable NHC employees and four (25-21) of the nonexcludable HC employees are not covered by the plan. All of these employees meet the eligibility rules of the qualified retirement plan yet are not covered under the plan. The plan will satisfy the safe harbor coverage test because it covers at least 70 percent of the NHC employees.

How well did you know this?
1
Not at all
2
3
4
5
Perfectly
15
Q

Johnson Brothers Co. has a noncontributory qualified profit sharing plan with 310 employees in total, 180 who are nonexcludable (40 HC and 140 NHC). The plan covers 72 NHC and 29 HC. The NHC receive an average of 4.5% benefit and the HC receive 6.5%. Which of the following statements is (are) correct?

  1. The Johnson Brothers Co. plan meets the ratio percentage test.
  2. The Johnson Brothers Co. plan fails the average benefits test.
  3. The plan must and does meet the Actual Deferral Percentage (ADP) test.

a) 1 only.
b) 2 only.
c) Both 1 and 2.
d) 1, 2, and 3.

A

Answer: C

The plan does not have to meet the ADP test because it is a noncontributory plan. The plan meets the ratio percentage test and fails the average benefits test.

How well did you know this?
1
Not at all
2
3
4
5
Perfectly
16
Q

What are the the coverage tests for qualified retirement plans?

A
How well did you know this?
1
Not at all
2
3
4
5
Perfectly
17
Q

Felipe’s Fine Furniture has 105 employees. Ninety of the employees are nonexcludable and 15 of those are highly compensated (75 are nonhighly compensated). The company’s qualified profit-sharing plan benefits 8 of the highly compensated employees and 40 of the nonhighly compensated employees. Does the profit-sharing plan sponsored by Felipe’s Fine Furniture meet the coverage test?

a) Yes, the plan meets the average benefits percentage test.
b) Yes, the plan meets the general safe harbor test.
c) Yes, the plan meets the ratio percentage test.
d) Yes, the plan meets ratio percentage test and the general safe harbor test.

A

Answer: C

The plan meets the ratio percentage test. The percentage of NHC employees covered by the plan is 53.33 percent and the percentage of HC employees covered by the plan is 53.33%. The ratio percentage of the NHC employees covered by the plan compared to the ratio percentage of the HC employees covered by the plan is 100 percent (53.33% / 53.33%) which is greater than the ratio requirement of at least 70 percent.

Shortcut

Another method of determining whether the plan meets the ratio percentage test is to determine the minimum number of nonexcludable NHC employees that must be covered by the plan to pass the ratio percentage test. This can be determined by calculating 70 percent of the percentage of HC covered by the plan multiplied by the number of nonexcludable NHC employees. In this problem, it would be calculated as follows: [((8 / 15) x 70%) x 75] = 28. Twenty-eight NHC employees must be covered to pass the ratio percentage test. The facts do not give us any information to determine if the plan meets the average benefits percentage test. The plan does not meet the general safe harbor test which requires that at least 70 percent of the NHC employees are covered by the plan.

How well did you know this?
1
Not at all
2
3
4
5
Perfectly
18
Q

What are the required coverage levels for various numbers of employees in the Defined Benefit 50/40 test?

A
How well did you know this?
1
Not at all
2
3
4
5
Perfectly
19
Q

Safeguard-It Company has a defined benefit plan with 200 nonexcludable employees (40 HC and 160 NHC). They are unsure if they are meeting all of their testing requirements. What is the minimum number of total employees that must be covered on a daily basis to conform with the requirements set forth in the IRC?

a) 40.
b) 50.
c) 80.
d) 100.

A

Answer: B

The 50/40 rule requires that defined-benefit plans cover the lesser of 50 employees or 40 percent of all eligible employees. Here 40 percent would be 80, so 50 is less than 80. This would be the absolute minimum number of covered employees.

How well did you know this?
1
Not at all
2
3
4
5
Perfectly
20
Q

Big River Bank has a defined benefit plan with 60 employees. What is the minimum number of employees the defined benefit plan must cover to conform with the requirements set forth by the IRC?

a) 24.
b) 30.
c) 42.
d) 50.

A

Answer: A

The plan must cover the lesser of 50 people or 40 percent of all employees. In this case, the lesser would be 40 percent of 60, or 24 people.

How well did you know this?
1
Not at all
2
3
4
5
Perfectly
21
Q

Which of the following statements concerning choosing the most appropriate type of vesting schedule for a qualified plan-restrictive vs. generous is (are) correct?

  1. Two advantages of choosing a restrictive vesting schedule are (1) to reduce costs attributable to employee turnover and (2) to help retain employees.
  2. Three advantages of choosing a liberal vesting schedule in which there is immediate and full vesting are (1) to foster employee morale, (2) keep the plan competitive in attracting employees, and (3) to meet the designs of the small employer who desires few encumbrances to participation for the “employee family.” a

) 1 only.

b) 2 only.
c) Both 1 and 2.
d) Neither 1 nor 2.

A

Answer: C

Both Statements 1 and 2 are correct.

How well did you know this?
1
Not at all
2
3
4
5
Perfectly
22
Q

Which of the following vesting schedules may a non-top-heavy profit-sharing plan use?

  1. 2-to-6-year graduated.
  2. 3-year cliff.
  3. 1-to-4-year graduated.
  4. 3-to-7-year cliff.

a) 1 only.
b) 2 and 3.
c) 1, 2, and 3.
d) 1, 2, 3, and 4.

A

Answer: C

As a result of the PPA 2006, a profit-sharing plan must vest at least as rapidly as a 3-year cliff or 2-to-6 year graduated schedule without regard to the plan’s top-heavy status. The profit-sharing plan can follow any vesting schedule that provides a more generous vesting schedule.

How well did you know this?
1
Not at all
2
3
4
5
Perfectly
23
Q

Amparo is a key employee participant in a top-heavy profit sharing plan which follows the least generous graduated vesting schedule permitted under PPA 2006. Each year of her five-year employment with Mystic Mountain Resort, she has received an employer contribution equal to $10,000 to her profit-sharing plan account. Today the balance of her profit-sharing plan is $65,000. If Amparo terminated employment with Mystic Mountain Resort today, what is the vested balance of her profit-sharing plan?

a) $39,000.
b) $52,000.
c) $55,000.
d) $65,000.

A

Answer: B

Amparo’s vested balance in the profit-sharing plan account is $52,000. Under PPA 2006, the least generous graduated vesting schedule permitted for a profit-sharing plan is a 2-to-6 year graduated vesting schedule. The fact that the plan is top-heavy does not impact the vesting schedule under PPA 2006. Therefore, Amparo is 80% vested in the contributions to the account. There were no employee deferral contributions to the plan. Thus, 80% of $65,000 = $52,000.

How well did you know this?
1
Not at all
2
3
4
5
Perfectly
24
Q

Which of the following employees is a key employee for 2021?

  1. Mark, an officer of the company, who earns $100,000 per year and owns two percent of the company.
  2. Melissa, who earns $13,000 per year and owns five percent of the company.
  3. Tina, an officer of the company who earns $200,000.
  4. Jean, a 10 percent owner of the company who earns $4,000 per year as a secretary.

a) 4 only.
b) 3 and 4.
c) 2 and 3.
d) 1, 3, and 4.

A

Answer: B

Only Tina and Jean are considered key employees. A key employee is anyone who is any one or more of the following: (1) a greater than five percent owner, or (2) a greater than one percent owner with compensation in excess of $150,000, or (3) an officer with compensation in excess of $185,000 (2021).

How well did you know this?
1
Not at all
2
3
4
5
Perfectly
25
Q

The qualified profit-sharing plan of Super Spa, LLP is considered top-heavy for the year. Julia, a plan participant, has annual compensation equal to $65,000 and Julia is not a key employee. What is the minimum amount Super Spa, LLP must contribute on behalf of Julia?

a) $1,300
b) $1,950
c) $2,050
d) $2,400

A

Answer: B

The minimum amount that Super Spa, LLP must contribute to the qualified profit-sharing plan on Julia’s behalf is $1,950 ($65,000 x 3%) assuming that they contribute at least three percent to key employees.

How well did you know this?
1
Not at all
2
3
4
5
Perfectly
26
Q

A company’s defined benefit pension plan utilizes a funding formula that considers years of service and average compensation to determine the pension benefit payable to the plan participants. If Brenda is a participant in this defined benefit pension plan and she has 30 years of service with the company and average compensation of $75,000, what is the maximum pension benefit that can be payable to Brenda at her retirement?

a) $18,000.
b) $54,000.
c) $75,000.
d) $230,000.

A

Answer: C

The maximum amount payable from a defined benefit pension plan is the lesser of $230,000 (2021) or 100 percent of the average of the employee’s three highest consecutive years compensation. Because the average of Brenda’s compensation is $75,000, she would be limited to receiving a pension benefit at her retirement of $75,000.

How well did you know this?
1
Not at all
2
3
4
5
Perfectly
27
Q

What are the characteristics of Defined Benefit Pension Plans and Defined Contribution Pension Plans?

A
How well did you know this?
1
Not at all
2
3
4
5
Perfectly
28
Q
A
How well did you know this?
1
Not at all
2
3
4
5
Perfectly
29
Q

Which of the following is (are) a defined benefit plan formula(s)?

a) Unit benefit (a.k.a. percentage-of-earnings-per-year-of-service) formula.
b) Flat-percentage formula.
c) Flat-amount formula.
d) All of the above.

A

Answer: D

All of the above are benefit formulas used by defined benefit plans.

How well did you know this?
1
Not at all
2
3
4
5
Perfectly
30
Q

Tip!

A

Cash balance plans are a popular choice to get rid of old expensive DB plans!

How well did you know this?
1
Not at all
2
3
4
5
Perfectly
31
Q

All of the following statements concerning cash balance pension plans are correct EXCEPT:

a) The cash balance plan is generally motivated by two factors: selecting a benefit design that employees can more easily understand, and as a cost-saving measure.
b) The cash balance plan is a defined benefit plan.
c) The cash balance plan has no guaranteed annual investment return to participants.
d) The cash balance plan is subject to minimum funding requirements.

A

Answer: C

A basic component of a cash balance plan is the guaranteed minimum investment return.

How well did you know this?
1
Not at all
2
3
4
5
Perfectly
32
Q

Generally, which of the following are noncontributory plans?

  1. 401(k) and money purchase pension plans.
  2. 401(k) and thrift plans.
  3. Thrift plans and ESOPs.
  4. Money purchase pension plans and profit-sharing plans.

a) 4 only.
b) 1 and 2.
c) 3 and 4.
d) 1, 2, 3, and 4.

A

Answer: A

Employers generally contribute to money purchase pension plans, ESOPs, and profit-sharing plans. Employees contribute (thus contributory plans) to 401(k)s and thrift plans.

How well did you know this?
1
Not at all
2
3
4
5
Perfectly
33
Q

What is the difference between earned income and unearned income?

A

Earned Income:

  • W-2 income
  • Schedule C net income
  • K-1 income from an LLC
  • K-1 income from a partnership where the partner is a material participant
  • Alimony (If divorce agreement was signed prior to or by 12/31/18. TCJA 2017)

Not Earned Income:

  • Earnings and profits from property, such as rental income, interest income, and dividend income
  • Capital gains
  • Pension and annuity income
  • Deferred compensation received (compensation payments postponed from a past year)
  • Income from a partnership for which you do not provide services that are a material income-producing factor
  • Any amounts excluded from income, such as foreign income and housing costs Alimony from a divorce agreement signed after 12/31/2018.
  • Unemployment benefits
  • Investment returns as a limited partner in a partnership
  • Income flowing from an S-corporation via Schedule K-1
  • Social Security benefits
  • Worker’s compensation
How well did you know this?
1
Not at all
2
3
4
5
Perfectly
34
Q

Which of the following vesting schedules may a top-heavy qualified profit-sharing plan use?

a) 1-to-5-year graduated.
b) 5-year cliff.
c) 3-to-7-year graduated.
d) 4-to-8-year graduated.

A

Answer: A

As a result of the PPA 2006, qualified profit-sharing plans must use a vesting schedule that pro-vides participants with vested benefits at least as rapidly as either a 2-to-6 year graduated vesting schedule or a 3-year cliff vesting schedule. This requirement applies without regard to whether the profit-sharing plan is a top-heavy plan. Options B, C, and D all vest less rapidly than the required schedule.

How well did you know this?
1
Not at all
2
3
4
5
Perfectly
35
Q

Which entities may establish a 401(k) plan?

A
How well did you know this?
1
Not at all
2
3
4
5
Perfectly
36
Q

What are the characteristics of a Roth 401(k) account?

A
How well did you know this?
1
Not at all
2
3
4
5
Perfectly
37
Q

A non-safe harbor 401(k) plan allows plan participants the opportunity to defer taxation on a portion of regular salary simply by electing to have such amounts contributed to the plan instead of receiving them in cash. Which of the following statements are rules that apply to 401(k) salary deferrals?

  1. Salary deferral into the 401(k) plan are limited to $19,500 for individuals younger than 50 for 2021.
  2. A nondiscrimination test called the actual deferral percentage test applies to salary deferral amounts.

a) 1 only.
b) 2 only.
c) Both 1 and 2.
d) Neither 1 nor 2.

A

Answer: C

Both Statements 1 and 2 are correct.

How well did you know this?
1
Not at all
2
3
4
5
Perfectly
38
Q

What is the Actual Deferral Percentage (ADP) Test, and how is it calculated?

A
How well did you know this?
1
Not at all
2
3
4
5
Perfectly
39
Q

Hard Rock Construction sponsors a 401(k) profit-sharing plan. In the current year, Hard Rock Construction contributed 25 percent of each employees’ compensation to the profit-sharing plan. The ADP of the 401(k) plan for the NHC was 3.5 percent. If Jeff, age 57, earns $100,000 and is a six percent owner, what is the maximum amount that he may defer into the 401(k) plan for this year?

a) $5,500.
b) $12,000.
c) $19,500.
d) $26,000.

A

Answer: B

Jeff is highly compensated because he is more than a five percent owner, so the maximum that he can defer to satisfy the ADP test requirements is 5.5 percent (3.5% + 2%) and because he is over 50, he can defer the additional $6,500 (2021) as a catch-up contribution. Jeff can defer $5,500 (5.5% x $100,000) and $6,500 (the catch-up) for a total of $12,000.

How well did you know this?
1
Not at all
2
3
4
5
Perfectly
40
Q

Golden Reef Spa has 325 employees (300 NHC and 25 HC). Of these employees, 300 are non-excludable (275 NHC and 25 HC). If 208 of these NHC are covered under the Golden Reef Spa qualified profit-sharing plan, and 25 of these HC are covered under the Golden Reef Spa qualified profit-sharing plan, with certainty, which of the following coverage tests does Golden Reef Spa pass?

a) Safe Harbor Test.
b) Ratio Percentage Test.
c) Average Benefits Test.
d) Both the Safe Harbor Test and the Ratio Percentage Test.

A

Answer: D

Both the Safe Harbor Test and the Ratio Percentage Test are certainly passed. The plan covers 208 of the nonexcludable NHC employees which is 75.6%-greater than the 70% required to pass the Safe Harbor Test. The ratio of the NHC covered to the HC covered is 75.6% (75.6% / 100%), so the plan passes the Ratio Percentage Test also. The information does not provide the average benefit percentages of the employees to determine whether the plan passes the Average Benefits Test.

How well did you know this?
1
Not at all
2
3
4
5
Perfectly
41
Q

Which of the following clauses in a 401(k) plan can assist the plan in meeting the requirements of the ADP test?

a) Attestation clause.
b) No-Contest clause.
c) Negative election clause.
d) Deferral plan clause.

A

Answer: C

A negative election clause can assist a 401(k) plan in meeting the ADP test because it automatically deems that an employee defers a specific amount unless he elects out of the automatic deferral amount. Answers A and D do not exist and answer B is a clause commonly found in a will.

How well did you know this?
1
Not at all
2
3
4
5
Perfectly
42
Q

Which of the following is true regarding negative elections?

  1. A negative election is a provision whereby the employee is deemed to have elected a specific deferral unless the employee specifically elects out of such election in writing.
  2. Negative elections are no longer approved by the IRS.
  3. When an employer includes a negative election in its qualified plan, the employer must also provide 100% immediate vesting.

a) 1 only.
b) 1 and 3.
c) 2 and 3.
d) 1, 2, and 3.

A

Answer: A

Negative elections are approved by the IRS, and they are available for both current and new employees. Negative elections do not require 100% immediate vesting. All employee contributions, however, are 100% vested.

How well did you know this?
1
Not at all
2
3
4
5
Perfectly
43
Q

Retirement plan participants who wish to take advantage of a qualified plan’s loan provision must agree to the following restrictions on the amount of the loan and how it is repaid:

a) A participant may borrow no more than $50,000, or one-half of the vested account balance, whichever is less.
b) A participant’s loan must be repaid within 5 years, unless the loan is used to acquire a participant’s principal residence.
c) A participant is allowed to repay the loan any time within the 5-year period.
d) Both A and B.

A

Answer: D

Option C is incorrect. It describes “sham” repayments. Sham repayments are the reason that the rules were designed so that the $50,000 loan limit is reduced by the highest outstanding loan balance during the one-year period ending the day before the loan date. The rules also added a restriction that requires level amortization of loan repayments of principal and interest being made at least quarterly.

44
Q

Shredders, a shop that specializes in confetti has a 401(k) plan that allows plan loans up to the legal limit allowed by law. Participants must repay the loans under the most generous repayment schedule available by law. Shredders plan has the attached employee information. Which of the following statements is correct?

a) If Jill repays her $19,000 loan by the end of 20x1, she may take a loan of $50,000 any time in 20x2.
b) Diana can borrow $100,000 (50% of her vested account balance) from her account.
c) The maximum Laura can borrow from her account is $8,000.
d) John could borrow $17,500 from his plan for the purchase of a personal residence, but he would have to repay the loan within ten years.

A

Answer: C

Laura can borrow up to $10,000 from her 401(k) plan. Since her loan balance is $2,000, she can borrow an additional $8,000. The maximum loan is the lesser of 50% of a participant’s vested account balance, or $50,000, reduced by the highest outstanding loan balance within the last 12 months. Thus, the maximum loan Jill could take would be $50,000 less $19,000, or $31,000. Diana can only borrow $50,000. John will not have to repay the loan within 10 years because the loan proceeds are being used for a home purchase.

45
Q

What are the similarities and differences between stock bonus plans and profit-sharing plans?

A
46
Q

Drew, age 61, is a participant in a stock bonus plan. The value of the employer stock contributions to the plan over the course of his participation totaled $165,000. On December 1, 20x1, Drew takes a full distribution of the employer stock from the plan at a value of $550,000. Four-teen months later, Drew sells all of the stock for $400,000. Which of the following statements is true?

a) Drew has a long-term capital gain of $385,000 for 20x1.
b) Drew has ordinary income of $165,000 in 20x1.
c) Drew has a long-term capital loss of $235,000 in 20x2.
d) Drew has ordinary income of $165,000 and a long-term capital gain of $385,000 in 20x1.

A

Answer: B

Because Drew is taking a lump-sum distribution from a qualified plan of employer stock, he will not have to recognize the net unrealized appreciation until he disposes of the employer stock. At the time of the distribution, however, the value of the stock, as of the date of contribution to the plan, will be taxable as ordinary income. Any gain on the subsequent sale of the stock will be taxable as long-term capital gain. In this case, Drew will recognize $165,000 of ordinary income at the date of the distribution (20x1) and long-term capital gain of $235,000 ($400,000 - $165,000) at the date of sale (20x3).

47
Q

On January 15, of last year, Sean retired from Peyton, Inc. with 1,000 shares of Peyton, Inc. stock in his stock bonus plan. Peyton, Inc. took deductions equal to $20 per share for the contributions made on Sean’s behalf. At retirement, Sean took a lump-sum distribution of the employer stock. The FMV of the stock at distribution was $35 per share. On July 15 of this year, Sean sold the stock for $40 per share. Which of the following will Sean report on his tax return when he sells the Peyton, Inc. stock?

a) $0 of ordinary income and short-term capital gain, $20,000 of long-term capital gain.
b) $20,000 of ordinary income, $20,000 of short-term capital gain.
c) $35,000 of ordinary income, $5,000 long-term capital gain.
d) $40,000 of ordinary income.

A

Answer: A

Sean will not be subject to ordinary income at the date the stock is sold. At the date the stock was distributed to Sean, $20,000 ($20 x 1,000) would have been subject to ordinary income tax. At the date of sale, Sean would have had $0 of short-term capital gain (the gain after the date of distribution) and $20,000 of long-term capital gain (NUA of $15,000 plus LTCG of $5,000 based on the time from distribution to sale of the stock).

48
Q

What are the requirements for ESOPs to qualify for nonrecognition of gain treatment?

A
  • The ESOP must own at least 30 percent of the corporation’s stock immediately after the sale.
    • The seller or sellers must reinvest the proceeds from the sale into qualified replacement securities within 12 months after the sale and hold such securities three years.
  • Qualified replacement securities are securities in a domestic corporation, including stocks, bonds, debentures, or warrants, which receive no more than 25 percent of their income from passive investments. The qualified replacement securities can be in the form of stock in an S Corporation.
    • The corporation that establishes the ESOP must have no class of stock outstanding that is tradable on an established securities market.
    • The seller or sellers, relatives of the seller or sellers, and 25 percent shareholders in the corporation are precluded from receiving allocations of stock acquired by the ESOP through the rollover.
    • The ESOP may not sell the stock acquired through the rollover transaction for three years.
    • The stock sold to the ESOP must be common or convertible preferred stock and must have been owned by the seller for at least three years prior to the sale.
  • If the seller purchases and retains qualified replacement securities, there will be no taxable event.
49
Q

Gordon owns Advertising Solutions, Inc. (ASI) and sells 100% of the company stock on July 1 of the current year to an ESOP for $3,000,000. Gordon had an adjusted basis in the ASI stock of $450,000. If Gordon reinvests in qualified replacement securities before the end of the current year, which of the following statements is true?

a) Gordon will not recognize long-term capital gain or ordinary income in the current year.
b) Gordon must recognize $2,550,000 of long-term capital gain in the current year.
c) Gordon must recognize $450,000 of ordinary income in the current year.
d) If Gordon dies before selling the qualified replacement securities, his heirs will have an adjusted taxable basis in the qualified replacement securities of $450,000, Gordon’s carryover adjusted basis.

A

Answer: A

A major advantage for an ESOP is the ability of the owner to diversify his interest in a closely held corporation. In this case, if Gordon reinvests in qualified replacement securities within 12 months of the sale to the ESOP, he will not recognize capital gain or ordinary income on the sale to the ESOP. If Gordon dies the heirs will receive the securities with an adjusted taxable basis equal to the FMV at Gordon’s date of death or the alternate valuation date.

50
Q

Which of the following is/are elements of an effective waiver for a pre-retirement survivor annuity?

  1. The waiver must be signed within six months of death.
  2. The waiver must be signed by a plan participant.
  3. The waiver must be signed by the nonparticipant spouse and notarized or signed by a plan official.

a) 3 only.
b) 1 and 2.
c) 2 and 3. d

) 1, 2, and 3.

A

Answer: A

Only the nonparticipant spouse must sign the waiver.

51
Q

On April 30, Ava, age 42, received a distribution from her qualified plan of $150,000. She had an adjusted basis in the plan of $500,000 and the fair market value of the account as of April 30 was $625,000. Calculate the taxable amount of the distribution and any applicable penalty.

a) $30,000 taxable, $3,000 tax penalty.
b) $30,000 taxable, $0 tax penalty.
c) $120,000 taxable, $12,000 tax penalty.
d) $150,000 taxable, $15,000 tax penalty.

A

Answer: A

Because the distribution to Ava does not qualify for the exception to the 10 percent penalty, the taxable amount of the distribution will be subjected a 10 percent penalty. To calculate the amount of the distribution that is return of adjusted basis, the adjusted basis in the plan is divided by the fair market value of the plan as of the day of the distribution. This ratio is then multiplied times the gross distribution amount. As such, $120,000 [($500,000 / $625,000) x $150,000] of the $150,000 distribution is return of adjusted taxable basis. Accordingly, $30,000 ($150,000 - $120,000) will be subject to income tax, and there will be a $3,000 ($30,000 x 10%) tax penalty.

52
Q

What are the exceptions to the 10% early withdrawal penalty for qualified plans?

A

For Qualified Plans to avoid the 10% penalty, they make a “Child MESS AT DQ Exception”

  • Child: $5,000 per taxpayer for Birth or Legal Adoption
  • Medical expenses that exceed 7.5 percent of AGI
  • Substantially equal periodic payments (Section 72(t))
  • Attainment of age 55 and separation from service OR Qualified public safety employee who separates from service after age 50
  • Federal tax levy
  • Death OR Disability
  • Qualified Domestic Relations Order (QDRO)
  • Exception if the distributions are dividends paid within 90 days of the plan year end from an ESOP.
53
Q

What are the exceptions to the 10% early withdrawal penalty for IRAs?

A

For IRAs to avoid the 10% penalty, they say “HIDE ME Tax Child before 59 1/2”

  • first time home purchase up to $10,000
  • health insurance for unemployed
  • death OR disability
  • higher education
  • medical expenses in excess of 7.5% of AGI
  • substantially equal periodic payments (section 72)
  • federal tax levy
  • child: birth or legal adoption
  • age 59 1/2
54
Q

Which of the following qualified plan distributions will be subjected to a 10% early withdrawal penalty?

a) Ray, age 35, takes a $100,000 distribution from his profit-sharing plan to pay for his son’s college tuition.
b) Debra, age 56, was terminated from UBEIT Corporation. Debra takes a $125,000 distribution from the UBEIT retirement plan to pay for living expenses.
c) Frank, age 47, takes a $1,000,000 distribution from his employer’s profit-sharing plan. Six weeks after receiving the $800,000 check (reduced for 20% withholding), Frank deposited $1,000,000 into a new IRA account.
d) Marie, age 22, begins taking equal distributions over her life expectancy from her qualified plan. The annual distribution is $2,000.

A

Answer: A

The distribution described in option A will be subjected to the 10% penalty. Education expenses are only an exception to the 10% penalty for IRAs, not qualified plans. All of the other options are exceptions to the 10% early withdrawal penalty. Option B describes the exception for separation from service after age 55. Option C describes the exception for the rollover of qualified plan assets. Option D describes the exception for substantially equal periodic payments.

55
Q

Which of the following qualified plan distributions are subject to a 10% early withdrawal penalty?

  1. Debra, age 56, currently employed by UBEIT Corporation, takes a $125,000 distribution from the UBEIT 401(k) plan.
  2. Frank, age 60, takes a $1,000,000 distribution from his employer’s profit-sharing plan. Ten days after receiving the $800,000 check (reduced for 20% withholding), Frank deposited the $800,000 into a new IRA account.
  3. Marie, age 22, withdraws $2,000 of her contributions from her 401(k).

a) 1 only.
b) 3 only.
c) 2 and 3.
d) 1 and 3.

A

Answer: D

Situation 1 is subjected to the 10% early withdrawal penalty because Debra has not separated from service. Situation 2 will not be subjected to the 10% early withdrawal penalty because Frank is older than 59 1⁄2. Situation 3 will be subjected to the 10% penalty because Marie does not qualify for any of the exclusions from the 10% penalty.

56
Q

Mario turned 72 in November of this year (2021). He was a participant in his employer’s profit-sharing plan. His profit-sharing plan had an account balance of $250,000 on December 31 of this year, and $200,000 on December 31 of last year. When must Mario take his first required minimum distribution?

a) December 31, 2021.
b) April 1, 2022.
c) December 31, 2022.
d) April 1, 2023.

A

Answer: B

Mario must take a required minimum distribution before April 1, 2022, the year after he attains the age of 72.

57
Q

In August of 2021, Mekhi turned 72. He was a participant in his former employer’s profit-sharing plan. His profit-sharing plan had an account balance of $600,000 on December 31, 2020, and $450,000 on December 31, 2019. According to the uniform lifetime table the factors for ages 71, 72, and 73 are 26.5, 25.6, and 24.7 respectively. What is the amount of Mekhi’s first required minimum distribution for 2021?

a) $0.
b) $16,981.
c) $17,578.
d) $23,437.50.

A

Answer: D

$600,000 divided by 25.6 = $23,437.50. His distribution for 2021 must be taken by April 1, 2022. His 2021 distribution is based on year-end 2020 balance and the factor for his age on December 31, 2020. His 2022 distribution will be based on the December 31, 2021, balance and his age factor for 73, and must be distributed by December 31, 2022. Delaying his first distribution causes two distribution to be completed in one year.

58
Q

Dita began taking required minimum distributions from her profit-sharing plan 7 years ago. In 2021, Dita died after suffering a heart attack. She had not named a beneficiary of her profit-sharing plan. Which of the following statements is false?

a) Dita’s estate may take a full distribution of the profit-sharing plan’s assets in the year of her death.
b) In the year of Dita’s death the minimum required distribution will be equal to the minimum required distribution had Dita not died.
c) Dita’s estate must take a distribution of the profit-sharing plan account balance by the end of 2021.
d) The required minimum distribution for 2022 will be calculated utilizing the factor according to Dita’s age reduced by one.

A

Answer: C

Because Dita had already begun required minimum distributions, the five-year requirement does not apply, but her estate must continue taking distributions over Dita’s remaining life expectancy reduced by one each year. All of the other options are true statements.

59
Q

What are the options for required minimum distributions following a participant’s death before 12/31/19?

A
60
Q

What are the options for required minimum distributions following a participant’s death after 12/31/19?

A
61
Q

What are the characteristics of qualified plans for plan selection purposes?

A
62
Q

What’s the process for selecting a qualified retirement plan?

A
63
Q

Jay, age 55 and the owner of a bicycle repair shop, has come to you to establish a qualified plan. The repair shop, which employs mostly young employees, has had steady cash flows over the past few years, but Jay foresees shaky cash flows in the future as new bicycle prices decline. Jay would like to allocate as much of the plan contributions to himself as possible. He is the only employee whose compensation is in excess of $100,000. Which of the following qualified plans would you advise Jay to establish?

a) Profit-sharing plan.
b) Defined benefit pension plan.
c) Cash balance pension plan.
d) Money purchase pension plan (Integrated).

A

Answer: A

A profit-sharing plan would be the best choice for Jay’s company. All of the other options described pension plans that require mandatory funding. A pension plan would not be an appropriate choice due to the company’s unstable cash flows

64
Q

Generally, younger entrants are favored in which of the following plans?

  1. Defined benefit pension plans.
  2. Cash balance pension plans.
  3. Target benefit pension plans.
  4. Money purchase pension plans.

a) 4 only.
b) 2 and 4.
c) 1 and 3.
d) 2, 3, and 4.

A

Answer: B

Cash balance and money purchase pension plans favor younger entrants. Defined benefit and target benefit pension plans favor older age entrants with less time to accumulate, and therefore, require higher funding levels.

65
Q

Which of the following statements regarding determination letters for qualified plans is true?

a) When a qualified plan is created, the plan sponsor must request a determination letter from the IRS.
b) An employer who adopts a prototype plan must request a determination letter from the IRS.
c) If a qualified plan is amended, the plan sponsor must request a determination letter from the Department of Labor.
d) A qualified plan which receives a favorable determination letter from the IRS may still be disqualified at a later date.

A

Answer: D

Determination letters are issued by the IRS at the request of the plan sponsor. The plan sponsor is not required to request a determination letter. Even if the determination letter is requested and approved, the IRS may still disqualify the plan.

66
Q

Laura has self-employment income of $87,500 for the year. How much is her self-employment tax?

a) $10,747
b) $11,637
c) $12,363
d) $13,387

A

Answer: C

The calculation is $87,500 x 0.9235 x 0.153 = $12,363. The full 15.3% can be used to save a step due to the income being under the social security wage base.

67
Q

Dimitri operates Downtown Discount Pharmacy, a sole proprietorship. Downtown Discount Pharmacy sponsors a profit-sharing plan. Dimitri had net income of $205,000 and paid self-employment taxes of $23,197 for the year. If Dimitri makes a 15% of salary contribution on behalf of all of his employees to the profit-sharing plan, how much is the contribution to the profit-sharing plan on behalf of Dimitri?

a) $22,642.
b) $25,219.
c) $29,010.
d) $30,750.

A

Answer: B

$205,000 Net Income <11,599> Less: 1/2 SE tax ($23,197 x 50%) $193,401 Net SE Income x 0.1304 0.15/1.15 $25,219 Dimitri can only contribute to himself what he contributes to his employees.

68
Q

Patrick, age 60, is a member of Worksalot, LLC. Worksalot sponsors a profit-sharing plan. Patrick’s portion of the net income was $250,000 and one-half of his self-employment taxes were $12,201 for this year. If Worksalot makes a 25% of salary contribution on behalf of all of its employees to the profit-sharing plan, how much is the contribution to the profit-sharing plan on behalf of Patrick?

a) $47,560.
b) $50,000.
c) $48,780.
d) $58,000.

A

Answer: A.

250,000 Net Income

<12,201> Less: 1/2 SE tax (from above)

= 237,799 Net SE Income

x 0.20 [0.25/1.25]

= $47,560

69
Q

What are the Prohibited Transactions in a qualified plan?

A

A disqualified person is any of the following:

  1. A fiduciary of the plan.
  2. A person providing services to the plan.
  3. An employer, any of whose employees are covered by the plan.
  4. An employee organization, any of whose members are covered by the plan.
  5. Any direct or indirect owner of 50% or more of any of the following:
    • The combined voting power of all classes of stock entitled to vote, or the total value of shares of all classes of stock of a corporation that is an employer or employee organization described in (3) or (4).
    • The capital interest or profits interest of a partnership that is an employer or employee organization described in (3) or (4).
    • The beneficial interest of a trust or unincorporated enterprise that is an employer or an employee organization described in (3) or (4).
  6. A member of the family of any individual described in (1), (2), (3), or (5). (A member of a family is the spouse, ancestor, lineal descendant, or any spouse of a lineal descendant.)

Prohibited transactions generally include actions by a disqualified person that potentially could have adverse consequences to the plan or participants, including:

  • Transfer of plan income or assets to, use of them by, or for the benefit of a disqualified person.
  • Self-dealing by a fiduciary.
  • Receipt of consideration by a fiduciary for his own account when working with a party dealing with the plan (e.g., attorney, accountant).
  • Selling, exchanging, leasing, buying as well as lending or borrowing between a disqualified person and the plan.
70
Q

All of the following are acceptable reasons for an employer to terminate a qualified retirement plan except:

a) The employer is no longer in a financial position to make further plan contributions.
b) The employer no longer wants to maintain the plan because it must cover other employees other than just himself.
c) The plan benefits are not meaningful amounts, and participants are limited in their ability to make deductible IRA contributions.
d) To lower plan costs and ease administrative complexity, the employer wants to switch plan designs.

A

Answer: B

Retirement plans must not be created as a tax shelter for the owner. If they have been, plan termination can result in retroactive disqualification. All other statements are acceptable reasons to terminate a qualified retirement plan.

71
Q

Each of the following are requirements imposed by law on qualified tax-advantaged retirement plans EXCEPT:

a) Plan documentation.
b) Employee vesting.
c) Selective employee participation.
d) Employee communications.

A

Answer: C

Broad employee participation, as opposed to selective participation, is a requirement of a tax-advantaged retirement plan. All of the others are requirements for “qualified” plans.

72
Q

Andrew, age 53, had the following items of income:

  • Investment returns as a limited partner in a partnership of $1,200.
  • Unemployment compensation of $350.
  • Income from a law practice of $600.
  • Deferred compensation from a former employer of $14,000, not constructively received this year.
  • Alimony of $750 (he was divorced in 2017).
  • Wages of $1,000.

What is the maximum contribution Andrew can make to an IRA for this year?

a) $1,750.
b) $2,350.
c) $2,700.
d) $5,000.

A

Answer: B

Andrew is limited to making an IRA contribution equal to the lesser of $7,000 (2021) (including the catch-up) or his earned income for the year. Andrew’s earned income includes his law practice income, alimony, and wages, which total $2,350.

73
Q

Hamilton, a single 29-year-old, deferred 2% of his salary, or $2,000, into a 401(k) plan sponsored by his employer during the current year. What is the maximum deductible IRA contribution Hamilton can make during the current year?

a) $0.
b) $1,100.
c) $4,400.
d) $5,500.

A

Answer: A

Hamilton cannot make a deductible IRA contribution for the year because he is an active participant in a qualified plan with an AGI of at least $100,000 ($2,000 / 2%), which exceeds the single taxpayer phase-out limits for 2021.

74
Q

Rex, a married 29-year-old, deferred 10% of his salary, or $10,000, into a 401(k) plan sponsored by his employer this year. His wife was unemployed all year and did not receive unemployment compensation. Assuming Rex has no other income, what is the maximum contribution Rex’s wife can make to her Roth IRA for this year?

a) $0.
b) $1,000.
c) $4,500.
d) $6,000.

A

Answer: D

Rex’s wife can make a $6,000 Roth IRA contribution, the maximum for this year, because Rex’s AGI of $100,000 ($10,000 deferral divided by 10% deferral percentage) is below the phase-out limit of $198,000. Even though she does not have any earned income of her own, she can use Rex’s earnings to qualify for the contribution. The Roth IRA has contribution limitation based on AGI.

75
Q

Darryl, single and age 54, is a participant of his employer’s qualified profit-sharing plan. For the current year, he received a forfeiture allocation of $25, but the employer did not make any other contribution for the year. Darryl would like to make a deductible IRA contribution. If Darryl’s AGI is $80,000 (all comprised of W-2 earnings and portfolio income), what is the maximum deductible IRA contribution Darryl may make?

a) $0.
b) $3,000.
c) $5,500.
d) $6,500.

A

Answer: A

Because of the forfeiture allocation Darryl received from his employer’s plan, he would be considered an active participant of his employer’s qualified plan. Accordingly, his maximum deductible contribution to the IRA may be limited based upon his AGI. The AGI phaseout for a single active participant in a qualified plan is $66,000 - $76,000 (2021). Since Darryl’s AGI exceeds the threshold, he cannot make a deductible IRA contribution. He can make a non-deductible contribution, sp ensure you double-check what the question asks for.

76
Q

What are the exceptions to the 10 percent withdrawal penalty?

A

QRPs and IRAs

  • Death
  • Attainment of age 59 1⁄2
  • Disability
  • Substantially equal periodic payments (Section 72(t))
  • Medical expenses that exceed 7.5% of AGI (7.5% was made permanent by the Taxpayer Certainty and Disaster Tax Relief Act of 2020)
  • $5,000 per taxpayer for the birth or legal adoption of a child
    • The child must not be the child of your spouse; the child must be under age 18 or over the age of 18 if they are unable to care for themselves.

Only QRPs

  • Qualified Domestic Relations Order (QDRO)
    • Where there is a distribution at divorce and the payee is under 59 1/2, the use of a QDRO-directed distribution will result in a taxable event, but will not incur the 10% early withdrawal penalty. Under the same circumstances except that the distribution is from an IRA, the result is both a taxable event and the application of the 10% early withdrawal penalty. However, the payee in any case can choose to rollover the distribution in which case the rollover rules would apply or the payee can take substantially equal periodic payments under section 72(t).
  • Attainment of age 55 and separation from service
  • Public safety employee who separates from service after age 50

Only IRAs

  • Higher education expenses
  • First-time home purchase (up to $10,000)
  • Payment of health insurance premiums by unemployed
77
Q

Sam, age 54 and single, has compensation this year of $85,000. His employer does not sponsor a qualified plan, so Sam would like to contribute to a Roth IRA. What is Sam’s maximum contribution for this year to the Roth IRA?

a) $0.
b) $$6,000.
c) $$7,000.
d) $$26,000.

A

Answer: C

The maximum Roth IRA contribution is $6,000 plus $1,000 (2021) for those individuals 50 and over. Sam can make a $$7,000 contribution to his Roth IRA. Sam is not phased out (Single Roth IRA phase-out for 2021 is $125,000 - $140,000).

78
Q

At the age of 57, Julian converted his traditional IRA, valued at $45,000, to a Roth IRA. At age 60, Julian took a distribution from this Roth IRA of $100,000 to buy a new car for his daughter for college. Which of the following statements is true with regards to the distribution from the Roth IRA?

a) $100,000 will be subjected to ordinary income tax.
b) $55,000 will be subjected to ordinary income tax.
c) $55,000 will not be subjected to ordinary income tax or penalty.
d) $55,000 will be subjected to ordinary income tax and penalty.

A

Answer: B

For a distribution from a Roth IRA to be a qualified distribution, the distribution must be on account of death, disability, the owner attaining the age of 591⁄2, or the first-time purchase of a home, AND the distribution must occur five years after the account was created. In this case, since the Roth was not created five or more years before the distribution, the distribution will be taxable to the extent it represents earnings in the account, $55,000. Since the distribution was taken after Julian attained 59 1⁄2, it will not be subjected to the 10% penalty.

79
Q

What is the first year in which a single taxpayer, age 58 in 20x1, could receive a qualified distribution from a Roth IRA, if he made a $4,000 contribution to the Roth IRA on April 1, 20x2, for the tax year 20x1?

a) 20x3.
b) 20x4.
c) 20x5.
d) 20x6.

A

Answer: D

A qualified distribution can only occur after a five-year period has occurred and is made on or after the date on which the owner attains age 591⁄2, made to a beneficiary or the estate of the owner on or after the date of the owner’s death, attributable to the owner’s being disabled, or for a first-time home purchase. The five-year period begins at the beginning of the taxable year of the initial contribution to a Roth IRA. The five-year period ends on the last day of the individual’s fifth consecutive taxable year beginning with the taxable year described in the preceding sentence. Therefore, the first year in which a qualified distribution could occur is 20x6

80
Q

What investments aren’t allowed in IRAs — and what are the exceptions?

A

Life insurance and collectibles are not allowed in an IRA.

Gold, silver, platinum, or palladium are permitted.

81
Q

What are the prohibited IRA transactions?

A

If an individual or beneficiary of an IRA engages in ANY of the following transactions, then the account will cease to be an IRA as of the first day of the current taxable year:

  • Selling, exchanging, or leasing of any property to an IRA;
  • Lending money to an IRA;
  • Receiving unreasonable compensation for managing an IRA;
  • Pledging an IRA as security for a loan;
  • Borrowing money from an IRA; or
  • Buying property for personal use (present or future) with IRA funds.

If a “deemed distribution” is made due to a prohibited transaction then the entire balance in the IRA is treated as having been distributed. In this case, the taxpayer will be subject to ordinary income tax on the entire balance and will also be subject to the 10 percent early withdrawal penalty.

82
Q

Which of the following statements regarding a SEP is true?

  1. The maximum contribution to a SEP is the lesser of 100% of compensation or $58,000 for 2021.
  2. A SEP is appropriate for an employer with many part-time employees who want to limit coverage under the SEP.
  3. Contributions to a SEP must vest at least as rapidly as a 5-year cliff vesting schedule or 2-to-6-year graduated vesting schedule.
  4. If a partnership makes a flat percentage contribution equal to 25% of all employees’ salary for the year to a SEP, a partner earning $100,000 during the year would receive a $25,000 contribution.

a) 4 only.
b) 1 and 3.
c) 1 and 4.
d) None of the statements are true.

A

Answer: D

None of the statements are true. The maximum contribution to a SEP is the lesser of 25% of compensation or $58,000 for 2021. A SEP is inappropriate for employers with many part-time employees who want to limit coverage under the SEP as the SEP requires coverage after a short period of time. Contributions to a SEP are always 100% vested. A partner is considered self-employed and, therefore, subject to the special calculation for self-employed individuals.

83
Q

Julia, age 55, is an employee of Spa de la Mer, Inc. Spa de la Mer sponsors a SEP IRA and would like to contribute the maximum amount to Julia’s account for the plan year. If Julia earns $14,000 per year from Spa de la Mer, what is the maximum contribution Spa de la Mer can make on her behalf to the SEP IRA?

a) $3,500.
b) $14,000.
c) $19,500.
d) $58,000.

A

Answer: A

Contributions to a SEP IRA are limited to the lesser of 25% of the employee’s compensation or $58,000 (2021). In this case, Julia’s compensation is $14,000, so the contribution on her behalf would be limited to 25% of $14,000, or $3,500.

84
Q

Which of the following are characteristics of a SIMPLE?

a) Contributions to a SIMPLE are 100% vested at all times.
b) The maximum contribution to a SIMPLE is the lesser of 25% of compensation or $58,000 for 2021.
c) A SIMPLE permits employer discretionary contributions.
d) A SIMPLE imposes a 25% penalty on distributions prior to 59 1⁄2.

A

Answer: A

Answer A is a correct statement. Answer B is the maximum contribution to a qualified plan. Answer C is incorrect; SIMPLEs require employer contributions for matching except for the first 2 years. Answer D is incorrect.

85
Q

Dahlia, age 54, earns $125,000 annually from Travelers Incorporated. Travelers sponsors a SIMPLE, and matches all employee deferrals 100% up to a 3% contribution. Assuming Dahlia defers the maximum to her SIMPLE, what is the total contribution to the account in the current year including both employee and employer contributions?

a) $13,500.
b) $17,250.
c) $20,250.
d) $25,750.

A

Answer: C

Dahlia can defer up to $16,500 ($13,500 deferral + $3,000 catch-up) because she is 50. Travelers’ match for Dahlia is 3% of her compensation, or $3,750 (3% x $125,000). The maximum contribution to Dahlia’s SIMPLE is $20,250 ($13,500 + $3,000 + $3,750).

86
Q

Nina, age 35, earns $200,000 annually from Travelers Incorporated. Travelers sponsors a SIM-PLE and matches all employee deferrals 100% up to a 3% contribution. What is the maximum employee deferral contribution to Nina’s SIMPLE account for this year?

a) $6,000.
b) $13,500.
c) $19,500.
d) $21,500.

A

Answer: B

Nina can defer up to $13,500, the maximum SIMPLE deferral. Travelers can match Nina up to 3% of her compensation, or $6,000 (3% x $200,000). The maximum contribution to Nina’s SIMPLE is $19,500 ($13,500 + $6,000), but the question asked for the maximum employee deferral — thus $13,500.

87
Q

Which of the following statements is (are) true?

  1. A SEP requires the plan sponsor to provide at least a 100% match up to 3% of all employee deferrals.
  2. A SEP plan can be established by employers who employ more than 100 employees who earn $5,000 or more during the preceding calendar year.
  3. SIMPLEs can be either contributory or noncontributory plans, whereas SEP plans are always noncontributory.
  4. An employer who wants to share the responsibility of retirement plan funding should establish a SIMPLE rather than a SEP.

a) 4 only.
b) 2 and 3.
c) 1, 2, and 3.
d) 2, 3, and 4.

A

Answer: A

Statements 1, 2, and 3 are incorrect statements. Statement 1 is incorrect because a SEP is a noncontributory plan which does not receive employee deferral contributions. Statement 2 is an incorrect statement as the 100-employee limit applies to SIMPLEs. Statement 3 is an incorrect statement as SIMPLEs are predominately contributory plans and SEPs are noncontributory plans. Statement 4 is correct because a SIMPLE is predominately a contributory plan with a fairly low employer contribution, thus sharing the burden of funding the employer and the employee.

88
Q

What are the similarities and differences among SIMPLE IRA, SIMPLE 401(k), and 401(k) plans?

A
89
Q

Which of the following accurately describes a 403(b) plan?

a) A 403(b) plan is a noncontributory qualified profit-sharing plan.
b) Because of catch-up provisions, the investment risk of the assets within a 403(b) plan is borne equally by the plan sponsor not the participant.
c) A participant’s deferral within a 403(b) plan will generally vest according to a 3-to-7-year graduated vesting schedule, however, a 5-year cliff vesting schedule may be used.
d) 403(b) plan assets can be invested indirectly in stocks and bonds through annuities or mutual funds.

A

Answer: D

Answer D is a correct statement accurately describing a 403(b) plan. Answer A is incorrect as a 403(b) plan is an employee deferral plan and is not a qualified plan. Answer B is incorrect as the investment risk is borne by the employee in all cases. Answer C is incorrect as an employee’s benefit within a 403(b) plan is always 100% vested.

90
Q

Which of the following statement(s) regarding 403(b) plans is true?

  1. Assets within a 403(b) plan may be invested in individual securities.
  2. A 403(b) plan usually provides a 3-to-7-year graduated vesting schedule.
  3. A 403(b) plan must pass the ACP test if it is an ERISA plan.
  4. In certain situations, a participant of a 403(b) plan can defer an additional $9,500 as a catch-up to the 403(b) plan.

a) 4 only.
b) 1 and 2.
c) 3 and 4.
d) 2, 3, and 4.

A

Answer: C

403(b) plan assets cannot be invested in individual securities, and deferrals to 403(b) accounts are always 100% vested. Statements 3 and 4 are true. Remember that 403(b) can have both the catch-up for age 50 & over ($6,500) and 15-year rule ($3,000).

91
Q

Which of the followings statement(s) regarding 403(b) plans is true?

  1. Assets within a 403(b) plan may be invested in annuities.
  2. Assets within a 403(b) plan may be invested in mutual funds.
  3. All 403(b) plans must pass the ADP test.
  4. In certain situations, a participant of a 403(b) plan can defer an additional $26,000 (catch-up) to a 403(b) plan in a single plan year.

a) 1 only.
b) 1 and 2.
c) 2 and 3.
d) 1, 2, and 4.

A

Answer: B

Only a contributory 403(b) plan has to pass the ADP test. 403(b) plans are not required to meet the ADP test, but 403(b) plans with employer contributions are required to meet the ACP test. The catch-up amount is $9,500 not $26,000.

92
Q

How are the three types of 457 plans related?

A
93
Q

Frank Fontana, age 42, has compensation of $72,000. The normal retirement age for his 457(b) plan is age 62. Frank has unused deferrals totaling $21,000 as of January 1, 2021. How much can Frank defer into his 457(b) public plan for 2021?

a) $19,500.
b) $22,500.
c) $26,000.
d) $39,000.

A

Answer: A

Frank is not within 3 years of the plan’s normal retirement age and, therefore, can only defer the normal $19,500. The $6,500 catch-up (2021) for those participants age 50 and over is not available because he is only 42 years old.

94
Q

Which of the following statements regarding 457 plans is (are) true?

  1. An individual who defers $19,500 to his 403(b) plan during 2021 can also defer $19,500 to a 457 plan during 2021 (salary and plan permitting).
  2. A 457 plan allows an executive of a tax-exempt entity to defer compensation into an ERISA-protected trust.
  3. In the final three years before normal retirement age, a participant of a government-sponsored 457 plan may be able to defer $39,000 (2021) for the plan year.

a) 1 only.
b) 2 only.
c) 1 and 3.
d) 2 and 3.

A

Answer: C

Statement 2 is false. The funds deferred to a 457 plan established for a tax-exempt entity do not have ERISA protection. Statement 1 is true as the deferral limits for 403(b) plans and 457 plans are separate. Statement 3 is true as certain 457 plans allow the participants to defer twice the annual deferral limit in the last three years before the plan’s normal retirement age.

95
Q

Which of the following statements concerning rabbi trusts is (are) correct?

a) A rabbi trust is a trust established and sometimes funded by the employer that is subject to the claims of the employer’s creditors, but any funds in the trust cannot generally be used by or revert back to the employer.
b) A rabbi trust calls for an irrevocable contribution from the employer to finance promises under a non-qualified plan, and funds held within the trust cannot be reached by the employer’s creditors.
c) A rabbi trust can only be established by a religious organization.
d) All of the above are correct.

A

Answer: A

Only Option A is correct as it describes a rabbi trust. Option B describes a secular trust. Option C is a false statement.

96
Q

What are the Characteristics of Alternative Deferred Compensation Arrangements?

A
97
Q

Which of the following statements concerning tax considerations of non-qualified retirement plans is (are) correct?

  1. Under IRS regulations an amount becomes currently taxable to an executive even before it is actually received if it has been “constructively received.”
  2. Distributions from non-qualified retirement plans are generally subject to payroll taxes.

a) 1 only.
b) 2 only.
c) 1 and 2.
d) Neither 1 nor 2.

A

Answer: A

Statement 2 is incorrect because payroll taxes are due on deferred compensation at the time the compensation is earned and deferred, not at the date of distribution. Statement 1 is a correct statement.

98
Q

What are the holding period and requirements for ISOs?

A

Requirements for Incentive Stock Option

  1. ISOs can only be granted to an employee of the corporation issuing the ISOs.
  2. The ISO plan must be approved by the stockholders of the issuing corporation.
  3. The ISOs must be granted within 10 years of the ISO plan date.
  4. The exercise of the ISO is limited to a 10-year period (5 years for 10%+ owners).
  5. At the date of the ISO grant, the exercise price must be greater than or equal to the fair market value of the stock.
  6. An ISO cannot be transferred except at death.
  7. An owner of more than 10% of a corporation cannot be given ISOs unless the exercise price is 110% of the fair market value at the date of grant and the option term is less than 5 years.
  8. The aggregate fair market value of ISO grants at the time the option is first exercisable must be less than or equal to $100,000 based on the grant price per year per executive. Any excess grant over the $100,000 is treated as a NQSO.*
  9. To qualify as an ISO, the executive must not dispose of the stock before the later of two years from the grant of the ISO or within one year of the exercise of the ISO.**
  10. The executive must be an employee of the corporation continuously from the date of the grant until at least three months prior to the exercise.
\* = tested occasionally
\*\* = tested often
99
Q

Tori receives ISOs with an exercise price equal to the FMV at the date of the grant of $22. Tori exercises these options 3 years from the date of the grant when the FMV of the stock is $30. Tori then sells the stock 3 years after exercising for $35. Which of the following statements is (are) true?

  1. At the date of grant, Tori will have ordinary income equal to $22.
  2. At the date of exercise, Tori will have W-2 income of $8.
  3. At the date of sale, Tori will have long-term capital gain of $13.
  4. Tori’s employer will not have a tax deduction related to the grant, exercise or sale of this ISO by Tori.

a) 3 only.
b) 3 and 4.
c) 2, 3, and 4.
d) 1, 2, and 4.

A

Answer: B

Statements 3 and 4 are correct. Since Tori held the underlying security 2 years from grant and one year from exercise before its sale, Tori will receive long-term capital gain treatment for the appreciation, and her employer will not have a deductible expense related to the ISO.

100
Q

Bailey received NQSOs with an exercise price equal to the FMV at the date of the grant of $22. Bailey exercises the options 3 years after the grant date when the FMV of the stock was $30. Bailey then sells the stock 3 years after exercising for $35. Which of the following statements is (are) true?

  1. At the date of the grant, Bailey will have ordinary income of $22.
  2. At the date of exercise, Bailey will have W-2 income of $8.
  3. At the date of sale, Bailey will have long-term capital gain of $5.
  4. Bailey’s employer will have a deductible expense in relation to this option of $22.

a) 3 only.
b) 2 and 3.
c) 2, 3, and 4.
d) 1, 2, 3, and 4.

A

Answer: B

Statements 2 and 3 are correct. Bailey would not have any taxable income at the date of grant provided the exercise price is equal to the fair market value of the stock. Bailey’s employer would receive a tax deduction equal to the amount of W-2 income: Bailey would be required to recognize $8 of W-2 income at the date of exercise. Bailey’s long-term capital gain is $5, calculated as the sales price of $35 less the exercise price of $30.

101
Q

Meredith was granted 100 NQSOs on January 12, 20x0. At the time of the option grant, the value of the underlying stock was $100, and the exercise price was equal to $100. If Meredith exercises the options on August 22, 20x4 when the stock is valued at $145, what are the tax consequences (per share) to Meredith?

a) $45 of W-2 income, $100 of short-term capital gain per share.
b) $100 of W-2 income, $45 of short-term capital gain per share.
c) $145 of W-2 income per share.
d) $45 of W-2 income per share.

A

Answer: D

At the exercise date of an NQSO, the individual will have to buy the stock at the exercise price and will have W-2 income for the appreciation of the stock value in excess of the exercise price. In this case, Meredith will have $45 ($145-$100) of W-2 income. There is no other gain or loss at exercise.

102
Q

What are the concepts related to ISOs and NQSOs?

A
103
Q

Emily works for M&S Law Firm. M&S pays (reimburses) Emily $265 per month for parking at work. How much of an income tax deduction does the law firm get related to the parking reimbursement?

a) $0.
b) $100.
c) $130.
d) $260.

A

Answer: A

No deduction for expenses associated with providing any qualified transportation fringe to employees is permitted after 2017. Emily can exclude the $265 from income.

104
Q

Which of the following situations would create an inclusion in an employee’s gross income in 2021?

a) Marsha is the director and manager of Holiday Hotel. As a condition of her employment, Marsha is required to live at the hotel. The value of this is $1,000 per month.
b) Jan is a secretary at JKL Law Firm. JKL provides her with free soft drinks. Jan estimates that she drinks $20 worth of soft drinks per month.
c) Greg is an airline pilot with We Don’t Crash Airlines, Inc. and is allowed to fly, as a passenger, for free on the airline whenever an open seat is available.
d) Peter moved from Houston to New Orleans. His expenses for the move included $400 of truck rental costs, $100 of lodging and $200 of pre-move house hunting expenses. Peter’s employer reimbursed him $600.

A

Answer: D

Peter must include $600 of the reimbursement in his gross income, as moving expenses are not excludible after 2017, except for active-duty military.

105
Q

Elisa is covered by a $200,000 group term life insurance policy of which her daughter is the sole beneficiary. Elisa’s employer pays the entire premium for the policy, for which the uniform annual premium is $0.75 per $1,000 per month of coverage. How much, if any, of the cost of the group term life insurance is excluded from Elisa’s gross income on an annual basis?

a) $0.
b) $450.
c) $1,350.
d) $1,800.

A

Answer: B

Elisa can exclude the cost of up to $50,000 of group term life insurance coverage. In this case, the cost of $50,000 of coverage is $450 [50 ($50,000 / $1,000) x 0.75 x 12 months].

106
Q
A